Set 1, Reumatology, Arpana

You might also like

Download as txt, pdf, or txt
Download as txt, pdf, or txt
You are on page 1of 8

1.

You are doing the annual review of a 50-year-old woman who has rheumatoid
arthritis. Which one of the following complications is most likely to occours as a
result of her disease?
A. Chronic ;umphocytic leukaemia
B. Hypertension
C. Colorectal cancer
D. Type 2 diabetes mellitus
E. Ischaemic heart disease
ANSWER: E

2. A 51-year-old male presents with an acute onset of swelling ans pain in his
right knee. Aspiration shows negatively birefringent crystals with no organisms
seen. His pain fails to settle with NSAIDs. What is the most appropriate next step
in his management?
A. Repeat joint aspiration and intra-articular depomedrol
B. Allopurinol
C. IV flucloxacillin
D. Diuretics
E. Low dosemethotrexate
ANSWER: A

3. Which of the following is least likely to be associated with ankylosing


spondylitis?
A. Apical fibrosis
B. Achilles tendonitis
C. Amyloidysis
D. Achalasia
E. Heart block
ANSWER: D

4. A 54-year-old man presents to the Emergency Department with a 2 day history of a


swollen, painful left knee. Aspirated joint fluid shows calcium pyrophosphate
crystals. Which of the following blood tests is most useful in revealing an
underlying cause?
A. Transferrin saturation
B. ACTH
C. ANA
D. Serum ferritin
E. LDH
ANSWER: A

5. Which one of the following is the most common ocular manifestation of rheumatoid
arthritis?
A. Scleritis
B. Episcleritis
C. Leratoconjunctivitis sicca
D. Corneal ulceration
E. Keratitis
ANSWER: C

6. Which one of the following drugs is least likely to cause gout?


A. Lithium
B. Bendrofluazide
C. Alcohol
D. Pyrazinamide
E. Furosemide
ANSWER: A

7. 39-year-old woman with a history of rheumatoid arthritis presents with a two day
history of a red right eye. There is no itch or pain. Pupils are 3mm, equal and
reactive to light. Visual acuity is 6/5 in both eyes. What is the most likely
diagnosis?
A. Keratoconjunctivits sicca
B. Scleritis
C. Glaucoma
D. Episcleritis
E. Anterior uveitis
ANSWER: D

8. A 64-year-old female is referred to rheumatology out-patients by her GP with a


history of arthritis in both hands. Which one of the following x-ray findings would
most favour a diagnosis of rheumatoid arthitis over other possible causes?
A. Loss of joint space
B. Periarticular osteopenia
C. Subchondral sclerosis
D. Osteophytes
E. Subchondral cysts
ANSWER: B

9. A 33-year-old man who is suspected pf having ankylosing spondylitis has a lumbar


spine x-ray. Which one of the following features is most likely to be present?
A. Wedge shaped discs
B. Sclerosis
C. 'Rugger-Jersey' spine
D. Osteophytes
E. Subchondral cysts
ANSWER: B

10. A 31-year-old patient is diagnosed with rheumatoid arthritis. Which of the


following of the folllowing is associated with a good prognosis?
A. Being a non-smoker
B Erosions on x-ray first developing 18 months after diagnosis
C. Sudden onset
D. Being diagnosed aged 35 years
E. Anti-CCP antibodies
ANSWER: C

11. Which one of the following statements regarding ankylosing spondylitis is


correct?
A. Schober's test assesses reduced chest expansion
B. HLA-B27 is positive in 50% of patients
C. Achilles tendonitis is a recognised association
D. It affects tendonitis is a recognised association
E. The typical age of presentation is between 40-50 years
ANSWER: C

12. Which one of the following conditions has polygenic inheritance?


A. Bartter's syndrome
B. Huntington disease
C. Ankylosing spondylitis
D. Fragile X syndrome
E. Von Willebrand's disease
ANSWER: C

13. Approximately what percentage of patients with psoriasis develop an associated


arthropathy?
A. 0.5%
B. 12-15%
C. 4-5%
D. 1%
E. 10%
ANSWER: E

14. You review a 48-year-old woman who is taking methotrexate for rheumatoid
arthritis. Concurrent prescription of which other medication should be avoided?
A. Erythromycin
B. Trimethoprim
C. Sumatriptan
D. Lansoprazole
E. Sodium Valproate
ANSWER: B

15. A 68-year-old presents with a painful swollen left knee which has failed to
settle after a weeks rest. There is no history of trauma. On examination he has a
moderate sized effusion. A plain radiograph is reported as follows: Some loss of
joint space. Liner calcification of arthicular cartilage. Whar is the most likely
diagnosis?
A. Pseudogout
B. Rheumatoid arthritis
C. Sarcoidosis
D. Gout
E. Osteoarthritis
ANSWER: A

16. A 69-year-old man presents with an acute episode of gout on his left first
metatarsal-phalangeal joint. What is the most likely underlying mechanism?
A. Sedentary lifestyle
B. Decreased renal excretion of uric acid
C. Increased endogenous production of uric acid
D. Starvation
E. Too much protein in diet
ANSWER: B

17. A 24-year-old female is investigated for intermittent pain and swelling of


metacarpal phalangeal joints for the past 3 months. An x-ray shows loss or joint
space and soft-tissue swelling. Rheumatoid factor is positive and a diagnosis of
rheumatoid arthritis is made. What is the most appropriate management to slow
disese progression?
A. Infliximab
B. Rituximab
C. Sulfasalazine
D. Methotrexate + sulfasalazine + short-course of prednisolone
E. Diclofenac
ANSWER: D

18. A 34-year-old kitchen worker presents with a two week history of pain in her
right wrist. She has recently emigrated from Ghana and has no past medical history
of note. On examination she is tender over the base os her right thumb and also
over the radial styloid process. Ulnar deviation of the wrist recreates the pain.
What is the most likely diagnosis?
A. Rheumatoid arthritis
B. Osteoarthritis of the carpometacarpal joint
C. De Quervain's tenosynovitis
D. Carpal tunnel syndrome
E. Systemic lupus erythematosus
ANSWER: C
19. A 54-year-old man is diagnosed as having gout. You are discussing ways to help
prevent future attacks. Which one of the following is most likely to precipitate an
attack of gout?
A. Chocolate
B. Brazil nuts
C. Eggs
D. Sardines
E. Smoking
ANSWER: D

20. A 45-year-old man present with a painful, swollen and red lefr middle toe.
There is no history of trauma and his symptoms have been present for around a week.
Which one of the following conditions is most associated with this presentations?
A. Diabetes mellitus
B. Systemic Sclerosis
C. Rheumatoid Arthritis
D. Bisphosphonate use
E. Psoriatic arthritis
ANSWER: E

21. A 25-year-old man presents with a painful, swollen left knee. He returned 4
week ago from a holiday in Spain. There is no history of trauma and he has no knee
problems previously. On examination he has a swollen, warm left knee with a full
range of movement. His ankle joints are also painful to move but there is no
swelling. On the soles of both feet you notice a waxy yellow rash. What is the most
likely diagnosis?
A. Rheumatoid arthritis
B. Psoriatic arthritis
C. Gout
D. Reactive arthritis
E. Gonococcal arthritis
ANSWER: D

22. A 24-year-old man is investigated for chronic back pain. Which one of the
following would most suggest a diagnosis of ankylosing spondylitis?
A. Reduced lateral flexion of the lumbar spine
B. Pain gets worse during the day
C. Accentuated lumhar lordosis
D. Pain on straight led raising
E. Loss of thoracic kyphosis
ANSWER: A

23. A 45-year-old man who is known to have haemochromatosis presents with a swollen
and painful right knee. An x-ray shows no fracture but extensive chondrocalcinosis.
Given the likely diagnosis, which one of the following is most likely to present in
the joint fluid?
A. Raised hyaluronic acid levels
B. Monosodium urare crystals
C. Bipyramidal oxalate crystals
D. Negatively birefringent calcium carbonate crystals
E. Positively birefringent rhomboid-shaped crystals
ANSWER: E

24. Which of the following statements is true regarding psoriatic arthropathy?


A. Skin disease always precedes joints disease
B. Approximately one-third of patients with psoriasis eventually develop
arthropathy
C. The mainstay of management is analgesia, physiotherapy and joint replacement
D. Males and females are equally affected
E. Arthritis mutilans is the most common subtype
ANSWER: D

25. A 71-year-old man presents with an erythematous, swollen first


metatarsophalangeal joint on the left foot. This is causing him considerable pain
and he is having difficulty walking. He has never had any previous similar
episodes. His past medical history includes atrial fibrillation and type 2 diabetes
mellitus and his current medications are warfarin, metformin and simvastatin. What
is the most appropriate treatment of this episode?
A. Intra-articular corricosteroid
B. Colchicine
C. Ibuprofen
D. Diclofenae
E. Prednisolone
ANSWER: B

26. Perinuclear antineutrophil cytoplasmic antibodies (pANCA) are most strongly


associated with which conditions?
A. Goodpasture's syndrome
B. Churg-Strauss syndrome
C. Polyarteritis nodosa
D. Wegener's granulomatosis
E. Autoimmune hepatits
ANSWER: B

27. A 41-year-old man with a past history of asthama presents with pain and
weakness in his left hand. Examinationfindings are consistent with a left ulnar
nerve palsy. Blood tests reveal an eosinophilia. Which one of the following
antobodies is most likely to be present>
A. ANA
B. Anti-Scl70
C. pANCA
D. Antipjospholipid antibodies
E. cANCA
ANSWER: C

28. A 44-year-old woman is seen in the rheumatology clinic. She has been referred
with Raynaud's phenomenon. During the revies of system she mentions that her GP is
organising an endoscopy to investigate dyspepsia. On examination she is noted to
have tight, shiny skin over her fingers. Which one of the following complications
she most likely to develop?
A. Bronchiectasis
B. Angiodysplasia
C. Arterial hypertension
D. Chronic kidney disease
E. Pulmanary hypertension
ANSWER: E

29. A 35-year-old woman who has severe Raynaud's disease is reviewed in clinic.
Three months age she was started on nifedipine. Unfortunately this has had a
minimal effect on her symptoms and has resulted in ankle oedema. What is the most
appropriate next step in management?
A. Aspirin
B. Sympathectomy
C. Intravenous prostacyclin
D. Methotrexate
E. Intravenens infliximab
ANSWER: C
30. Which one of the following is most likely to indicate an underlying connective
tissue disorder in a patient with Raynaud's phenomenon?
A. Chilblains
B. Bilateral symptoms
C. Female patient
D. Onset at 18 years old
E. Recurrent miscarriages
ANSWER: E

31. Which one of the following would not suggest an underlying connective tissue
disorder in a patient with Raynaud's?
A. Unilateral symptoms
B. Digital ulcers
C. Presence of autoantibodies
D. Onset at 25 Years old
E. Calcinosis
ANSWER: D

32. A 44-year-old female with a history of Raynaud's phenomenon is reveiwed in the


rheumatology clinic. She is currently being investigated for dysphagia. On
examination she is noted to have tight, shiny skin over her fingers. which one of
the following complications she most likely to develop?
A. Early onset dementia
B. Erythema nodosum
C. Malabsorption
D. Constrictive pericarditis
E. Erosive joint disease
ANSWER: C

33. Which one of the following is least associated with Behcet's syndrome?
A. Mouth ulcers
B. Genital ulcers
C. Conjunctivitis
D. Deep vein thrombosis
E. Aseptic meningitis
ANSWER: C

34. An Autoantibody screen reveals that a patient is positive for anti-Jo 1


antibodies. What is the most likely underlying diagnosis?
A. LImited cutaneous systemic sclerosis
B. Mixed connective tissue disease
C. Dematomyositis
D. Polymyositis
E. Diffuse cutaneous systemic sclerosis
ANSWER: D

35. Which one of the following is true regarding the investigation of a patient
with dermatomyositis?
A. Creatine kinase is characteristically normal
B. Muscle biopsy is contraindicated
C. Anti-Jo-1 antibodies are usually negative
D. Antinuclear antibodies are always negative
E. EMG is normal
ANSWER: C

36. A 54-year-old male presents with weakness of his upper arms. On examination he
is found to have a mscular rash over his back and the extensor aspects of his upper
arms. He is heavy smoker and his sodium is 121 mmol/l. What is the most likely
underlying diagnosis?
A. Addison's disease
B. Polymyositis
C. Overlap syndrome
D. Dermatomyositis
E. Hypothyroidism
ANSWER: D

37. A 41-year-old female presents with lethargy and pain all over her body. This
has been present for the past six months and is often worse when she is stressed or
cold. Clinical examination is unremarkable other than a large blood tests including
an autoimmune screen, inflammatory markers and thyroid function are normal. Given
the likely diagnosis, Which one of the following is not helpful in management?
A. Amitriptyline
B. Trigger point injections
C. Cognitive behavioural therapy
D. Exercise programme
E. paracetamol
ANSWER: B

38. A 41-year-old man presents with persistent fatigue for the past 8 months. Which
one of the following features is least consistent with a diagnosis of chronic
fatigue syndrome?
A. Dizziness
B. Painful lymph nodes without enlargement
C. Having a busy day improves the symptoms
D. Palpitaions
E. Headaches
ANSWER: C

39. Which one of the following features is least typical of polymyalgia rheumatica?

A. Elevated creatine kinase


B. Low-grade fever
C. Morning stiffness in proximal limb muscles
D. Polyarthralgia
E. Anorexia
ANSWER: A

40. A 61-year-old man is noted to have thickened patches of skin over his knuckles
and extensor surfaces consistent with Gottron's papules. His creatinine kinase
levels are also elevated. A diagnosis of dermatomyositis is suspected. Which one of
the following types of autoantibody is most specific for this condition?
A. Anti-scl-70 antibodies
B. Anti-Jo-l antibodies
C. Anti-nuclear antibodies
D. Anti-Mi-2 antibodies
E. Anti-centromere bodies
ANSWER: D

41. A 62-year-old man with lung cancer is suspected of having dermatomyositis.


Which one of the followinbg antibodies is most likely to be positive?
A. Anti-nuclear antibodies
B. Anti-centromere bodies
C. Anti-scl-70 antibodies
D. Anti-Jo-l antibodies
E. Anti-Mi-2 antibodies
ANSWER: A

42. Which one of the following is not a risk factor for developing osteoporosis?
A. Smoking
B. Obesity
C. Sedentary lifestyle
D. Premature menopause
E. Female sex
ANSWER: B

43. Which one of the following is least recognised as a risk factor for developing
osteoporosis?
A. Cushing's syndrome
B. Turmer's syndrome
C. Hyperparathyroidism
D. Hypothyroidism
E. Diabetes Mellitus
ANSWER: D

44. A 66-year-old female presents with pain at the base of her left thumb. She no
past medical history of note. On examination there is diffuse tenderness and
swelling of her left first carpometacarpal joint. What is the most likely
diagnosis?
A. Osteoarthritis
B. De Quervain's tenosynovitis
C. Gout
D. Rheumatoid arthritis
E. Primary hyperparathyroidism
ANSWER: A

45. A 66-year-old female is on long-term prednisolone therapy for polymyalgia


rheumatiea. What is the most appropriate protection against osteoporosis?
A. Hormone replacement therapy
B. Calcitonin
C. Oral Bisphosphonate
D. Calcium and Vitamin D
E. Hip-protectors
ANSWER: C

You might also like